23 votos

¿Cuándo hay suficientes Casimires?

Sé que un Casimir para un álgebra de Lie $\mathfrak{g}$ es un elemento central del álgebra envolvente universal. Por ejemplo, en $\mathfrak{so}(3)$ los generadores son los operadores de momento angular $J_1,J_2,J_3$ y un Casimir cuadrático es $\mathbf{J}^2$ .

Consideremos una representación irreducible $d$ de $\mathfrak{g}$ en $V$ . Por el Lemma de Schur cualquier Casimir $C$ se convierte en una matriz con un único valor propio $C_d$ . Si $d$ y $e$ son representaciones equivalentes, entonces $C_d=C_e$ ya que el cambio de base preserva los valores propios.

¿En qué circunstancias hay un número suficiente de Casimires como para que obtengamos la inversa de esta afirmación? En otras palabras, ¿cuándo podemos encontrar suficientes Casimires que $\{C_d,D_d,\dots F_d\}$ únicamente ¿etiquetas representaciones irreducibles? No sé cuántas necesitaríamos en general ni cómo demostrarlo.

Muchas gracias de antemano.

0 votos

La misma pregunta en Math.SE: math.stackexchange.com/q/288816/11127

23voto

Stefano Puntos 763

Aquí sólo discutiremos el caso de representaciones irreducibles finito-dimensionales (irreps) de un álgebra de Lie compleja semisimple $L$ .

Recordemos que el conjunto $Z$ de invariantes de Casimir es la centro $Z(U(L))$ de la álgebra envolvente universal $U(L)$ cf. por ejemplo este Correo de Phys.SE.

La pregunta de OP se responde sin pruebas en la p. 253 de la Ref. 1:

Teorema 2. Para toda álgebra de Lie semisimple $L$ de rango $r$ existe un conjunto de $r$ polinomio invariante del generador $t_a$ cuyos valores propios caracterizan las representaciones irreducibles de dimensión finita.

La Ref. 2 (que es uno de los libros más importantes sobre álgebras de Lie, al menos si uno está interesado en las pruebas) no se molesta en mencionar el Teorema 2 explícitamente. Sin embargo, es posible encadenar un conjunto de resultados más fundamentales (y sus pruebas) de la Ref. 2 para obtener el resultado buscado. A continuación esbozamos la estrategia de demostración.

Recordemos además que hay asociada una sistema radicular $\Phi$ al álgebra de Lie $L$ e imaginemos que hemos elegido una base $\Delta$ para $\Phi$ . La orden $|W|$ de la Grupo de Weyl $W$ es igual a las posibles elecciones de bases (desordenadas) e igual a las posibles elecciones de bases (fundamentales) Cámaras de Weyl .

Se demuestra en los capítulos 20-21 de la Ref. 2. que un irrep de dimensión finita tiene un único vector de mayor peso (único hasta la normalización) con alguna dominante integral peso $\lambda$ . A partir de ahora denotaremos tales irrep $V(\lambda)$ . (La Ref. 2. también define una noción de irrep $V(\lambda)$ cuando $\lambda$ es integral pero no dominante. Tales irreps son necesariamente de dimensión infinita, así que las ignoraremos). De ello se deduce que

Dos irreps $V(\lambda)$ y $V(\mu)$ son equivalentes (es decir, isomorfas) si sus pesos más altos son iguales $\lambda=\mu$ .

Como consecuencia de Teorema de Harish-Chandra el conjunto $Z$ de Casimires toma el mismo valor en dos irreps de mayor peso $V(\lambda)$ y $V(\mu)$ si $\lambda+\delta$ y $\mu+\delta$ pertenecen a la misma órbita de Weyl,

$$ \sigma(\lambda+\delta)~=~\mu+\delta, \qquad \sigma \in W. $$

Aquí $\delta$ es la mitad de la suma de las raíces positivas. Sin embargo, si ambos pesos integrales $\lambda$ y $\mu$ son dominantes, entonces $\lambda+\delta$ y $\mu+\delta$ deben pertenecer ambas a (el interior de) la cámara fundamental de Weyl, de modo que la reflexión de Weyl $\sigma={\bf 1}$ debe ser el elemento de identidad. En conclusión, obtenemos que

El conjunto $Z$ de Casimires toma el mismo valor en dos irreps de dimensión finita $V(\lambda)$ y $V(\mu)$ si sus pesos más altos son iguales $\lambda=\mu$ .

El teorema de Harish-Chandra se demuestra en el capítulo 23 de la Ref. 2. Véase también este y este entradas relacionadas con Math.SE.

Ejemplo: Consideremos el álgebra de Lie $L=sl(3,\mathbb{C})$ . El grupo de Weyl es $S_3$ . El álgebra de Lie $L$ tiene dos invariantes de Casimir independientes $C_2$ y $C_3$ ,

$$C_n ~:=~ {\rm str}({\rm ad} t_{a_1}\circ\ldots\circ{\rm ad} t_{a_n}) t^{a_1} \otimes\ldots\otimes t^{a_n}, \qquad n~\in~ \{2,3\}.$$

Consideremos la representación fundamental tridimensional $F$ y el dual/contragrediente representación $\bar{F}$ de $L$ que son irreps no equivalentes. Tienen pesos máximos $\lambda=(1,0)$ y $\mu=(0,1)$ respectivamente. En concreto, si $t_a$ , $a=1, \ldots, 8$ son generadores de $L=sl(3,\mathbb{C})$ (consejo de sombrero: Peter Kravchuk)

$$\bar{F}(t_a)~=~ -F(t_a)^t,$$

para que los Casimires $C_2$ (y $C_3$ ) toman el mismo valor (opuesto) en $F$ y $\bar{F}$

$$ {\rm tr}_{\bar{F}}\bar{F}(C_n)~=~(-1)^n{\rm tr}_{F}F(C_n), \qquad n~\in~ \{2,3\}. $$

Se puede demostrar que los valores son distintos de cero, de modo que las Casimires $C_2$ y $C_3$ distinguir entre los dos irreps no equivalentes $F$ y $\bar{F}$ como debe ser.

Referencias:

  1. A. O. Barut y R. Raczka, Teoría de representaciones de grupos y aplicaciones, 2ª ed., 1980.

  2. J.E. Humphreys, Introducción a las álgebras de Lie y a la teoría de la representación, (1980).

0 votos

Gracias, es maravilloso. Creo que ahora lo intuyo mucho mejor.

0 votos

@Qmechanic, ¿tienes alguna referencia para el hecho de que $C_3$ es el mismo en $F$ y $\bar{F}$ ? Simplemente no es obvio para mí, ya que obtenemos el dual por mapeo $\bar{F}(g)=-F(g)^T$ y IIRC $C_3$ es cúbica en generadores además de diagonal, por lo que $\bar{F}(C_3)=(-1)^3F(C_3)^T=-F(C_3)$ . ¿O es cero en ambos casos?

0 votos

@Qmechanic, he encontrado algunas fórmulas para los valores de $C_2$ y $C_3$ en representación con peso $(m_1,m_2)$ : $c_2=\frac{2}{3}(m_1^2+m_2^2+m_1 m_2+3m_1+3m_2)$ y $c_3=\frac{1}{9}(m_1-m_2)(2m_1+m_2+3)(2m_2+m_1+3)$ que conducen a $F(C_3)=\frac{20}{9}$ y $\bar{F}(C_3)=-\frac{20}{9}$ . Es de Lyahovsky-Bolokhov, pero dudo que exista una traducción al inglés. Mi afirmación en la respuesta eliminada también es de este libro (bueno, no fui preciso, la afirmación es que para álgebras simples Casimirs puede distinguir dos repeticiones no equivalentes cualesquiera). Sin embargo, todavía no he podido encontrar su referencia.

3voto

Brian B. Puntos 1519

Se puede demostrar (teorema de Racah) que el número de operadores de Casimir es igual al rango del álgebra (número de generadores que conmutan simultáneamente). Esto es cierto al menos para las álgebras semi-simples.

i-Ciencias.com

I-Ciencias es una comunidad de estudiantes y amantes de la ciencia en la que puedes resolver tus problemas y dudas.
Puedes consultar las preguntas de otros usuarios, hacer tus propias preguntas o resolver las de los demás.

Powered by:

X